An interesting inequality

a2+b2+c2ab+bc+ca+1389(ab+c+ba+c+ca+b)\large \frac{a^2+b^2+c^2}{ab+bc+ca}+\frac{1}{3}\geq\frac{8}{9}\bigg(\frac{a}{b+c}+\frac{b}{a+c}+\frac{c}{a+b}\bigg) Let a,b,c>0a,b,c>0, prove the above inequality.


This problem was taken from a discussion on VMF (Viet Maths Forum)
#Algebra

Note by P C
5 years ago

No vote yet
1 vote

  Easy Math Editor

This discussion board is a place to discuss our Daily Challenges and the math and science related to those challenges. Explanations are more than just a solution — they should explain the steps and thinking strategies that you used to obtain the solution. Comments should further the discussion of math and science.

When posting on Brilliant:

  • Use the emojis to react to an explanation, whether you're congratulating a job well done , or just really confused .
  • Ask specific questions about the challenge or the steps in somebody's explanation. Well-posed questions can add a lot to the discussion, but posting "I don't understand!" doesn't help anyone.
  • Try to contribute something new to the discussion, whether it is an extension, generalization or other idea related to the challenge.
  • Stay on topic — we're all here to learn more about math and science, not to hear about your favorite get-rich-quick scheme or current world events.

MarkdownAppears as
*italics* or _italics_ italics
**bold** or __bold__ bold

- bulleted
- list

  • bulleted
  • list

1. numbered
2. list

  1. numbered
  2. list
Note: you must add a full line of space before and after lists for them to show up correctly
paragraph 1

paragraph 2

paragraph 1

paragraph 2

[example link](https://brilliant.org)example link
> This is a quote
This is a quote
    # I indented these lines
    # 4 spaces, and now they show
    # up as a code block.

    print "hello world"
# I indented these lines
# 4 spaces, and now they show
# up as a code block.

print "hello world"
MathAppears as
Remember to wrap math in \( ... \) or \[ ... \] to ensure proper formatting.
2 \times 3 2×3 2 \times 3
2^{34} 234 2^{34}
a_{i-1} ai1 a_{i-1}
\frac{2}{3} 23 \frac{2}{3}
\sqrt{2} 2 \sqrt{2}
\sum_{i=1}^3 i=13 \sum_{i=1}^3
\sin \theta sinθ \sin \theta
\boxed{123} 123 \boxed{123}

Comments

Here's the solution. First, without losing generality, we can assume that ab+bc+ac=3ab+bc+ac=3, the problem'll become a2+b2+c2+183(ab+c+bc+a+ca+b)a^2+b^2+c^2+1\geq\frac{8}{3}\bigg(\frac{a}{b+c}+\frac{b}{c+a}+\frac{c}{a+b}\bigg) Now we'll do some manipulation on the variables in LHSLHS ab+c+bc+a+ca+b=a(a+b)(a+c)+b(b+a)(b+c)+c(c+a)(c+b)(a+b)(b+c)(c+a)\frac{a}{b+c}+\frac{b}{c+a}+\frac{c}{a+b}=\frac{a(a+b)(a+c)+b(b+a)(b+c)+c(c+a)(c+b)}{(a+b)(b+c)(c+a)} =a3+b3+c3+(a+b+c)(ab+bc+ca)(a+b+c)(ab+bc+ca)abc=(a+b+c)(a2+b2+c2abbcca+ab+bc+ca)+3abc3(a+b+c)abc=\frac{a^3+b^3+c^3+(a+b+c)(ab+bc+ca)}{(a+b+c)(ab+bc+ca)-abc}=\frac{(a+b+c)(a^2+b^2+c^2-ab-bc-ca+ab+bc+ca)+3abc}{3(a+b+c)-abc} =(a+b+c)(a2+b2+c2)+3abc3(a+b+c)abc=\frac{(a+b+c)(a^2+b^2+c^2)+3abc}{3(a+b+c)-abc} So now the inequality is a2+b2+c2+18[(a+b+c)(a2+b2+c2)+3abc]9(a+b+c)3abca^2+b^2+c^2+1\geq\frac{8[(a+b+c)(a^2+b^2+c^2)+3abc]}{9(a+b+c)-3abc} [9(a+b+c)3abc](a2+b2+c2+1)8(a+b+c)(a2+b2+c2)+24abc\Rightarrow [9(a+b+c)-3abc](a^2+b^2+c^2+1)\geq 8(a+b+c)(a^2+b^2+c^2)+24abc 9(a+b+c)(a2+b2+c2)+9(a+b+c)3abc(a2+b2+c2)3abc8(a+b+c)(a2+b2+c2)+24abc\Leftrightarrow 9(a+b+c)(a^2+b^2+c^2)+9(a+b+c)-3abc(a^2+b^2+c^2)-3abc\geq 8(a+b+c)(a^2+b^2+c^2)+24abc (a+b+c)(a2+b2+c2+9)3abc(a2+b2+c2+9)\Leftrightarrow (a+b+c)(a^2+b^2+c^2+9)\geq 3abc(a^2+b^2+c^2+9) a+b+c3abc\Rightarrow a+b+c\geq 3abc The last inequality can be proven easily by these following inequalities: (a+b+c)23(ab+bc+ca)(a+b+c)^2\geq 3(ab+bc+ca) and abc(ab+bc+ca3)3abc\leq\sqrt{\big(\frac{ab+bc+ca}{3}\big)^3}. The equality holds when a=b=ca=b=c

P C - 5 years ago
×

Problem Loading...

Note Loading...

Set Loading...